LSAT and Law School Admissions Forum

Get expert LSAT preparation and law school admissions advice from PowerScore Test Preparation.

 Administrator
PowerScore Staff
  • PowerScore Staff
  • Posts: 8916
  • Joined: Feb 02, 2011
|
#63983
Complete Question Explanation

Assumption. The correct answer choice is (D)

The author begins by providing that if theater managers do not believe that a film will generate
enough total revenue to make a profit (including food and beverage), they will not rent that film.
In other words, they must believe that a film is likely to be profitable. The greater the perceived
likelihood of profitability, then, the greater the chance that a manager will choose to rent a film.

..... Premise: managers more likely to rent :arrow: greater likelihood of profitability

The author then tells us about the producers’ belief: If they create movies that are attractive to
younger audiences, then the films will be more likely to be chosen by managers:

..... Conclusion: attractive to younger audiences :arrow: managers more likely to rent

The stimulus is followed by an assumption question, and the correct answer choice will need to
link previously unconnected pieces in a way that allows for the author’s conclusion to be logically
possible. So as a prephrase, think about what is necessary for theater managers to be more likely to
rent a film: if that film is likely to be profitable. So the connection of pieces should serve to satisfy
that required idea, where movies that are attractive to younger audiences have a higher probability of
being profitable:

..... Assumption: attractive to younger audiences greater likelihood of profit

That also works beautifully from an Assumption Negation standpoint, as negating it to show that
films attractive to younger audiences are NOT likely to be profitable would immediately invalidate
this argument: if we remove profitability then we also reduce the likelihood of theater managers
renting those films, which is the opposite of the author’s belief in the stimulus.

Answer choice (A): Although there is discussion of the role of likelihood of overall profitability,
the role played by concession stands is not specified, nor is the average consumption rate of adults
versus children and adolescents. Regardless, this choice fails to provide the needed link, so it is
incorrect.

Answer choice (B): There is no mention of whether movies that appeal to younger viewers might
also appeal to an older audience, and this is not an assumption on which the argument relies.
Answer choice (C): The author only provides that concession sales are part of the overall
profitability calculations, but offers no information about whether concessions or movie ticket sales
are more profitable. Further, this choice fails to provide the prephrased link between attractiveness to
younger viewers and likelihood of profitability.

Answer choice (D): This is the correct answer choice. This answer provides the needed link
between younger viewers and increased profits. The key assumption of conditionality is always that
the necessary condition is able to be fulfilled, as without it the sufficient cannot occur. In this case if
we know that films that appeal to younger audiences are more likely to be profitable than other films,
then the condition required for theater managers to rent those films (greater profit) is satisfied and
the author’s conclusion remains intact.

Answer choice (E): Like incorrect answer choice (B) above, this cannot be an assumption on
which the argument relies, because no information is provided regarding films that appeal to older
audiences (such films may or may not be the same films that appeal to younger crowds).
 ValVal
  • Posts: 8
  • Joined: Jul 05, 2017
|
#36969
Hi!
I actually get why answer (D) is correct, what troubles me is an explanation provided in LRB Workbook.
First of all, as I read the stimulus, I notate it this way:

Believe to generate the profit NOT :arrow: rent the film NOT, through contrapositive it gives me
Rent the film :arrow: believe to generate the profit.

Conclusion states: Make film attractive to young audience :arrow: managers rent the film

If I connect two new elements, it gives me: If you make film attractive to young audience, then managers believe it will generate profit, which answer choice (D) fits perfectly.

However, explanation offered in LRB Workbook states the first conditional premise as:

"Greater likelihood of profitability :arrow: managers more likely to rent" which is the reversal of how I diagrammed the premise.

Would you please clarify that moment for me?

Thank you!
Val
 Jon Denning
PowerScore Staff
  • PowerScore Staff
  • Posts: 904
  • Joined: Apr 11, 2011
|
#37624
Hi Val - thanks for the question! And well done: you've caught a mistake there, where the diagram in the text should indeed be reversed:

..... "Managers more likely to rent :arrow: Greater likelihood of profitability"

To me the key to this question is seeing that appealing to a younger audience presumably satisfies the necessary condition of greater profitability. The Assumption Negation Technique then does the trick nicely: if making films for a younger audience does not lead to profit, then managers wouldn’t be more likely to rent the films (first sentence) and the whole thing about a wider audience falls apart.

A lot of conditional Assumption questions work this way, where the underlying belief on the part of the author is that whatever condition is required for another to exist will be satisfied/met. That allows the thing that depends on it (the sufficient condition) to potentially come true.

So I think you've handled this one nicely! I've reported the error to our Publications Manager and will be sure it gets corrected in the next printing.

Thanks again!
User avatar
 Snomen
  • Posts: 35
  • Joined: Sep 30, 2021
|
#94833
Can anyone please provide a correct diagram for this question...? I am confused about how to correctly diagram it here.
Thank you
 Adam Tyson
PowerScore Staff
  • PowerScore Staff
  • Posts: 5153
  • Joined: Apr 14, 2011
|
#94861
The diagram in the official explanation at the top of the thread is correct. The question in this thread from 2017 was about an explanation in a printed version of the Logical Reasoning Bible, which has been corrected. Use the diagram here and you should be fine!
User avatar
 PresidentLSAT
  • Posts: 87
  • Joined: Apr 19, 2021
|
#100685
I found B to be way too attractive. Is it wrong because a movie that younger ppl find attractive can appeal to older people without yielding a profit?

Also, will the negation of this be that movies sometimes appeal to older people, making it an unnecessary assumption?
 Robert Carroll
PowerScore Staff
  • PowerScore Staff
  • Posts: 1787
  • Joined: Dec 06, 2013
|
#100719
PresidentLSAT,

Answer choice (B) is too extreme to have to be assumed. Why can't movies appeal to both types of audiences? We surely don't want all movies to appeal to all audiences, or otherwise the argument won't make much sense (why make films that theater managers consider attractive to younger audiences if ALL movies are attractive to everyone equally?), but there definitely can be pretty substantial overlap in appeal and the argument can still work.

The negation of answer choice (B) would be something like "Movies of the kinds that appeal to younger audiences also appeal to older audiences a fair amount of the time." And that negation is clearly not bad for the argument.

Robert Carroll

Get the most out of your LSAT Prep Plus subscription.

Analyze and track your performance with our Testing and Analytics Package.